Author Topic: #562 – Electroboom!  (Read 107885 times)

0 Members and 1 Guest are viewing this topic.

Offline bsfeechannelTopic starter

  • Super Contributor
  • ***
  • Posts: 1667
  • Country: 00
Re: #562 – Electroboom!
« Reply #25 on: November 07, 2021, 08:53:53 am »
I just have to ask: do you fundamentally understand the Maxwell-Faraday Equation? I have my doubts. Otherwise you would not claim that the circuit consists of only two resistors connected by wires that can be seen as "dead shorts" and will have no voltage across them.

Do you agree that the Maxwell-Faraday equation relates a time varying magnetic flux to an electric field? Do you agree that this changing flux _causes_ an electric field in the wire? Do you agree that an electric field is a difference in potential? Do you agree that a potential difference is measurable as a voltage between the wire ends?

OK. So let's suppose that the wires in Lewin's circuit have a 0.1 ohm resistance. R1=100 Ω and R2=900 Ω. So, around the loop, we will have 100 + 900 + 0.1 + 0.1 = 1000.2 ohms. Now let's suppose that the EMF generated by the varying magnetic field induces a current of, say, 1mA. Multiplying that current by the resistance of each wire (0.1 Ω) will give us 100 µV on them. The voltage on R1 will be 100 mV, while on R2 we will have 900 mV. Adding all voltages up, we will have 1.0002 V. So there you have it. The voltages around the loop will still not be adding up to zero.

If you don't believe the math. Build the circuit and measure those voltages. In fact, Cyriel Mabilde did exactly that and since the missing EMF didn't appear on the wires, he concocted the concept of the "masked EMF" that affects the probes of the meter only when you measure the voltages on the wires, but not on the resistors.  :-// This "masked EMF" is what would fool your meter and would sort of "mask" the missing EMF that is somewhere hidden in the wires.

Pseudo-science galore.

Quote
This is very much how a transformer works.

If you agree on all this, then you should be able to see that Dr. Lewins circuit is not just two resistors. It is in fact a transformer (or maybe generator) with the single secondary winding cut open in two places to insert resistors.


Yes. The secondary of a transformer is just a very low resistance wire shorting out a load. There are no other components in this kind of circuit. The wire and the load surround an area where you have a varying magnetic field. If you measure the voltage on the load and the voltage on the wire, they do not add up to zero.

Lewin used two resistors to enhance the effect, but he could very well have used just a resistor and a piece of wire. That would require a more sensitive scope to measure the voltage on the wire due to the low resistance, but it would obviously not be impossible. The voltages though would not still add up to zero.

Try it yourself. I encourage you. It will be fun to have your mind blown and a long cherished myth shattered to pieces. Things that look like black magic in electronics will suddenly start to make sense. You'll like it.

Quote
And it is the voltage across those two wire that you need to take into account when you add up all the voltages in the loop.

Yes I did and they still do not add up to zero.
« Last Edit: November 07, 2021, 09:10:24 am by bsfeechannel »
 

Offline jesuscf

  • Frequent Contributor
  • **
  • Posts: 499
  • Country: ca
Re: #562 – Electroboom!
« Reply #26 on: November 07, 2021, 03:38:46 pm »
OK. So let's suppose that the wires in Lewin's circuit have a 0.1 ohm resistance. R1=100 Ω and R2=900 Ω. So, around the loop, we will have 100 + 900 + 0.1 + 0.1 = 1000.2 ohms. Now let's suppose that the EMF generated by the varying magnetic field induces a current of, say, 1mA. Multiplying that current by the resistance of each wire (0.1 Ω) will give us 100 µV on them. The voltage on R1 will be 100 mV, while on R2 we will have 900 mV. Adding all voltages up, we will have 1.0002 V. So there you have it. The voltages around the loop will still not be adding up to zero.

Now let assume that the resistors in Lewin's circuit are of the wire bound kind.  Oopsie daisies!  As you can see we can assume lots of things.  But when electroboom did the experiment he accounted for many of this things... and KVL worked as expected:



One more thing:  bsfeechannel,  I called you out and you didn't deliver.  You have no idea of what you are talking about.  Talking to you is exactly like talking to a flat earther or an antivaxer.  So I decided to go thunderf00t on you call out all your BS. 

Homer: Kids, there's three ways to do things; the right way, the wrong way and the Max Power way!
Bart: Isn't that the wrong way?
Homer: Yeah, but faster!
 

Offline thinkfat

  • Supporter
  • ****
  • Posts: 2150
  • Country: de
  • This is just a hobby I spend too much time on.
    • Matthias' Hackerstübchen
Re: #562 – Electroboom!
« Reply #27 on: November 07, 2021, 05:02:01 pm »
I started writing a detailed response. But I think I let the posts of @bsfeechannel just stand on their own. I mean, just look at the calculations.

I conclude as follows:  :wtf: :palm: and will now stop  |O.
Have a nice day.
Everybody likes gadgets. Until they try to make them.
 

Offline jesuscf

  • Frequent Contributor
  • **
  • Posts: 499
  • Country: ca
Re: #562 – Electroboom!
« Reply #28 on: November 07, 2021, 05:10:52 pm »
I started writing a detailed response. But I think I let the posts of @bsfeechannel just stand on their own. I mean, just look at the calculations.

I conclude as follows:  :wtf: :palm: and will now stop  |O.
Have a nice day.

We have to keep calling him out and pointing out bsfeechannel's BS!  In a previous thread on the same subject about two/three years ago a snowflake got offended because I called him out (it may even have been bsfeechannel) so I stopped doing that.   That was a mistake.  BS must be pointed out and identified every time it shows up!  :box:
Homer: Kids, there's three ways to do things; the right way, the wrong way and the Max Power way!
Bart: Isn't that the wrong way?
Homer: Yeah, but faster!
 

Online bdunham7

  • Super Contributor
  • ***
  • Posts: 7812
  • Country: us
Re: #562 – Electroboom!
« Reply #29 on: November 07, 2021, 05:17:00 pm »
Yes. That's why we study vector calculus at any engineering graduation course before we study electromagnetism. Because to understand this bleep you need to think "fourth-dimentionally". I.e. you need to understand that electricity and magnetism are not phenomena confined to electronic components, and how this thing behave in space.

So, frame of references, relative positions, relative velocities, paths, rates of change. All of that counts.

That's three lines of blather that doesn't answer my question.  You're telling me that the display of my oscilloscope depends on not only how I connect it, but where I physically place it.  Although that may be true at some very minor level due to interference and other effects, it is nonsense here.  Suppose I had long cables on the two scopes and swapped their physical positions left and right, well out of reach of any magnetic field from the experiment.  Now they read the other way?  How about if I take the leads out perpendicularly and use two channels of one scope?  How about if I just use one scope?  These are just simple thought experiments that seem to me to reduce the experiment as claimed to an absurdity.  I know some people have looked at this experimentally and I have no comment on those arguments because I haven't looked at them closely.

Quote
The oscilloscopes "know" what branch they are measuring because they form a loop with each resistor. If you pause Lewin's presentation on Youtube ( /watch?v=nGQbA2jwkWI ) at 41:54, you'll see that the scope on the right forms a loop with R2. In that loop, there's no varying magnetic field. So, all the voltages will add up to zero according to Faraday's law and, in this case, to KVL, which is nothing more than a special case of Faraday's law when you have no varying magnetic field inside the path of the circuit. So the voltage on the right scope will have to be exactly the voltage on R2.

OK, if you are depending on the physical layout of the test leads, rather than the ultimate location of the oscilloscope (a much less ridiculous choice) than you need to reexamine the statement that there is no flux through that outer loop.  In order for the solenoid to induce a current in the loop, there has to be a net flux change inside the loop, but those flux lines have to eventually wrap around and go back to their opposite pole.  If they do that inside the loop, then they cancel out the net flux.  If they do that anywhere outside the loop, then your 'no varying magnetic field in the loop' becomes very questionable and would need to be measured by making another loop as physically close as possible but with a separate resistor not connected to the inner loop. 

Quote
Voltage can be path dependent if you are dealing with a non conservative electric field (i.e. one generated by a varying magnetic field).

Yes, I know the concepts and math (maybe once upon a time) when you are defining voltage potential as the work required to move a charge and so on.  There are multiple ways to define voltage, and when you factor in time variance, sometimes they don't add up.  If you are going to demonstrate a particular theory (path dependence) by relying on a test instrument, you need to consider what it is that the test instrument actually shows. 

As a thought experiment, envision an 'absolute' leaf electrometer that has one leaf and a charged plate is held parallel to the leaf and insulated from all surroundings.  The charged plate will generate a constant electric field which will cause the leaf to move according to the charge on itself.  Since the leaf will have a capacitance, the charge and voltage will be proportional and V = Q/C.  Thus I have a one-leaded absolute voltmeter.  It might need some calibrating, but it will now show the absolute (meaning without reference or referred to an electrically neutral object, not unsigned) voltage on its leaf without needing reference to anything else.  If I put two of these at different points in a circuit, I can observe both and then determine the difference.  If they are connected to the same point, then they have to show the same value because if not, current would flow from one to the other.  So if I put two of them at each point in the circuit, the ones that are connected to the same point must read the same simply because there's nothing to cause them to read otherwise.  That's all assuming there are no external electric or magnetic fields affecting the instruments--which is, of course a very problematic assumption in a time-variant system.

Now if you look at how most actual voltage measuring instruments work, whether they are an analog meter or oscilloscope, they measure the difference in absolute potential across their two input terminals.  They can do this either by reacting to electric fields directly or by allowing a small amount of current to flow.  In other words, the ideal voltmeter, however it works, reacts just like my hypothetical electrometer pair.   The voltmeter doesn't actually care about path dependence or anything else in the DUT, just about the potentials presented at its inputs.  It just measures the difference between two scalar quantities.

Quote
Since the wires have very low resistance compared to the resistors, they can be considered practically dead shorts.

Since this a time-variant system, the wires also have inductance.  I suspect that both the inductance and resistance are low enough in this case that they don't matter, but without numbers I can't say.  I think we can agree that the oscilloscopes read what they do because they are reading more or less the voltage drop across each resistor that results from the induced current.  Without examining Lewin's apparatus or experimenting myself, I couldn't say exactly how that was achieved.  A lot of the attempted explanations and experiments that have been shown regarding this seem as flawed as the original, but I'm pretty sure the answer is simply that there is another layer or two of complexity beyond the simple path-dependence that Lewin was demonstrating. 
A 3.5 digit 4.5 digit 5 digit 5.5 digit 6.5 digit 7.5 digit DMM is good enough for most people.
 

Online bdunham7

  • Super Contributor
  • ***
  • Posts: 7812
  • Country: us
Re: #562 – Electroboom!
« Reply #30 on: November 07, 2021, 05:29:24 pm »
Yes I did and they still do not add up to zero.

So this is why I am so reluctant to wade into this argument--I can't figure out what the dispute is.  It seems that the issue of why the oscilloscopes react the way they do in the example and 'voltages adding up' are somehow conflated? 

As for voltages adding up to zero, I don't know why anyone would think that they should.  So lets make a theoretical loop out of 100 1K resistors and apply a changing magnetic field strong enough to induce a 1mA current.  Now put 100 cheap voltmeters on each resistor (or measure the voltage thermally or whatever suits the imagination) and they should all read 1 volt, right?  Now pick a point and count around the circle, does it add up to zero or 100?  Why is this a question?  Or am I missing the point? 
A 3.5 digit 4.5 digit 5 digit 5.5 digit 6.5 digit 7.5 digit DMM is good enough for most people.
 

Offline rfeecs

  • Frequent Contributor
  • **
  • Posts: 807
  • Country: us
Re: #562 – Electroboom!
« Reply #31 on: November 07, 2021, 05:38:48 pm »
Wow.  The argument that never dies.

There (at least) two ways of modeling this:

Maxwell's equations model the electric and magnetic vector fields at every point in space.

Circuit theory uses a lumped circuit to model the voltages and currents.  KVL and KCL always hold in lumped circuit models.

Here are some great videos that explain it all:




 
The following users thanked this post: rsjsouza, bsfeechannel

Online bdunham7

  • Super Contributor
  • ***
  • Posts: 7812
  • Country: us
Re: #562 – Electroboom!
« Reply #32 on: November 07, 2021, 05:46:18 pm »
BS must be pointed out and identified every time it shows up! 

Perhaps, but it should be done with cogent and persuasive argument.  I'm not sure that is always happening here.  We could start by clarifying what we agree and disagree on as that seems to be not clear to me in this case.
« Last Edit: November 08, 2021, 12:08:00 am by bdunham7 »
A 3.5 digit 4.5 digit 5 digit 5.5 digit 6.5 digit 7.5 digit DMM is good enough for most people.
 

Offline thinkfat

  • Supporter
  • ****
  • Posts: 2150
  • Country: de
  • This is just a hobby I spend too much time on.
    • Matthias' Hackerstübchen
Re: #562 – Electroboom!
« Reply #33 on: November 07, 2021, 06:09:08 pm »
Please, don't say "a current is induced". Maxwell-Faraday clearly tells us that there is an electric field resulting from the changing magnetic flux. Current is what results out of Ohms law.
Everybody likes gadgets. Until they try to make them.
 

Online bdunham7

  • Super Contributor
  • ***
  • Posts: 7812
  • Country: us
Re: #562 – Electroboom!
« Reply #34 on: November 07, 2021, 07:07:30 pm »
Please, don't say "a current is induced". Maxwell-Faraday clearly tells us that there is an electric field resulting from the changing magnetic flux. Current is what results out of Ohms law.

Yeah, OK in this case where everything is being discussed and hotly disputed perhaps I should break that down into two steps.  But 'inducing a current' is hardly an unusual way of expressing the concept in general.
A 3.5 digit 4.5 digit 5 digit 5.5 digit 6.5 digit 7.5 digit DMM is good enough for most people.
 

Offline bsfeechannelTopic starter

  • Super Contributor
  • ***
  • Posts: 1667
  • Country: 00
Re: #562 – Electroboom!
« Reply #35 on: November 08, 2021, 12:35:32 am »
That's three lines of blather that doesn't answer my question.

The three lines of blather is not your answer. It's just a comment. You can ignore it if you will.

Quote
You're telling me that the display of my oscilloscope depends on not only how I connect it, but where I physically place it.

I'm not telling you. Nature is.

Quote
Although that may be true at some very minor level due to interference and other effects, it is nonsense here.

What you call the study of this very minor level interference and other effects is what is called the whole electronics engineering degree. And yes, it is difficult to understand.

Quote
Suppose I had long cables on the two scopes and swapped their physical positions left and right, well out of reach of any magnetic field from the experiment.  Now they read the other way?

That depends. With which of the resistors do your scope probes form a loop?

Quote
How about if I take the leads out perpendicularly and use two channels of one scope?
 

Perpendicularly to what? We are talking about a three dimensional space.

Quote
How about if I just use one scope?

You mean to measure each voltage separately? Please clarify.

Quote
These are just simple thought experiments that seem to me to reduce the experiment as claimed to an absurdity. I know some people have looked at this experimentally and I have no comment on those arguments because I haven't looked at them closely.

The problem with electromagnetism is that to have even a superficial understanding of it you have to go down a very deep rabbit hole, which most people don't care to do.

Quote
OK, if you are depending on the physical layout of the test leads, rather than the ultimate location of the oscilloscope (a much less ridiculous choice) than you need to reexamine the statement that there is no flux through that outer loop.

To be rigorous, you have to consider the position of the scope too. If it is inside the varying field, things change.

Quote
In order for the solenoid to induce a current in the loop, there has to be a net flux change inside the loop, but those flux lines have to eventually wrap around and go back to their opposite pole.  If they do that inside the loop, then they cancel out the net flux.  If they do that anywhere outside the loop, then your 'no varying magnetic field in the loop' becomes very questionable and would need to be measured by making another loop as physically close as possible but with a separate resistor not connected to the inner loop. 

Very pertinent question. Lewin took that precaution and demonstrated both theoretically and experimentally that the magnetic field outside the solenoid, where its length is much grater than its diameter, is negligible. See his lecture about it [ https://youtu.be/MXuZ1SRjpqk ]. So, from a practical point of view, the field outside the solenoid can be considered zero.

Quote
Now if you look at how most actual voltage measuring instruments work, whether they are an analog meter or oscilloscope, they measure the difference in absolute potential across their two input terminals.  They can do this either by reacting to electric fields directly or by allowing a small amount of current to flow.  In other words, the ideal voltmeter, however it works, reacts just like my hypothetical electrometer pair.   The voltmeter doesn't actually care about path dependence or anything else in the DUT, just about the potentials presented at its inputs.  It just measures the difference between two scalar quantities.

You are assuming your meter doesn't care about path dependence, but numerous experiments show otherwise. In Lewin's experiment, R1 and R2 are subject to electric fields of different intensities (assuming the resistors have the same size). And they can even be calculated. That's why the meters are showing different voltages (assuming there's no varying magnetic field in the loop made by the meter, the probes and the DUT).

If R1 and R2 were connected in parallel and attached to a battery, the voltages would be the same no matter what, because they would be subject to electric fields of the same intensity.

Stop for a moment and think. The voltages between points A and D in Lewin's experiment cannot be the same for the two resistors, otherwise they would have different currents flowing through them, which is impossible.


Quote
Since this a time-variant system, the wires also have inductance.  I suspect that both the inductance and resistance are low enough in this case that they don't matter, but without numbers I can't say.  I think we can agree that the oscilloscopes read what they do because they are reading more or less the voltage drop across each resistor that results from the induced current.  Without examining Lewin's apparatus or experimenting myself, I couldn't say exactly how that was achieved.  A lot of the attempted explanations and experiments that have been shown regarding this seem as flawed as the original, but I'm pretty sure the answer is simply that there is another layer or two of complexity beyond the simple path-dependence that Lewin was demonstrating.

Mehdi's experiment was spot on. His conclusion not quite.
 

Offline bsfeechannelTopic starter

  • Super Contributor
  • ***
  • Posts: 1667
  • Country: 00
Re: #562 – Electroboom!
« Reply #36 on: November 08, 2021, 01:48:08 am »
Wow.  The argument that never dies.

Every trade has its fair share of myths and in all cases they perpetuate because they are uttered by popularizers. We can't be naïve to think that we will manage to reverse the spreading of those myths, but I think it is always important to debunk them when necessary, so people can see the truth and decide if they will accept it or not.

Quote
There (at least) two ways of modeling this:

Maxwell's equations model the electric and magnetic vector fields at every point in space.

Circuit theory uses a lumped circuit to model the voltages and currents.  KVL and KCL always hold in lumped circuit models.

Lewin's circuit is simply unlumpable, because you cannot confine the varying magnetic field somewhere outside the path of the circuit.

Quote
Here are some great videos that explain it all:

Great. Indeed. The theory, the experiments and even the numerical simulations agree and show that KVL doesn't hold under a varying magnetic field. I saw the comments for the first video and I was glad to see that people repeated the experiment, checked the theory and confirmed that this myth is what it is: a myth.

Some even are casting doubts about Mehdi's integrity, which is a good thing. In science the only word of authority is that of nature's.
 

Offline bsfeechannelTopic starter

  • Super Contributor
  • ***
  • Posts: 1667
  • Country: 00
Re: #562 – Electroboom!
« Reply #37 on: November 08, 2021, 01:52:27 am »
I started writing a detailed response. But I think I let the posts of @bsfeechannel just stand on their own. I mean, just look at the calculations.

What is wrong with the calculations?
 

Offline bsfeechannelTopic starter

  • Super Contributor
  • ***
  • Posts: 1667
  • Country: 00
Re: #562 – Electroboom!
« Reply #38 on: November 08, 2021, 02:21:54 am »
But when electroboom did the experiment he accounted for many of this things... and KVL worked as expected:

Nope. Lewin used the theory to predict that the two resistors would have different voltages. Mehdi' experiment showed that Lewin's prediction was right. Mehdi admitted that explicitly.

Then Mehdi said that although the experiment matched exactly what Lewin predicted, he, Mehdi, thought that Lewin's was wrong.

This is a case of doublethink, where someone simultaneously accepts two mutually contradictory beliefs as correct, often in contravention to one's own sense of reality.

Quote
One more thing:  bsfeechannel,  I called you out and you didn't deliver. 

Not interested in your piss contest.

Quote
You have no idea of what you are talking about.  Talking to you is exactly like talking to a flat earther or an antivaxer.  So I decided to go thunderf00t on you call out all your BS.

Be my guest.

 

Offline jesuscf

  • Frequent Contributor
  • **
  • Posts: 499
  • Country: ca
Re: #562 – Electroboom!
« Reply #39 on: November 08, 2021, 02:51:12 am »
But when electroboom did the experiment he accounted for many of this things... and KVL worked as expected:

Nope. Lewin used the theory to predict that the two resistors would have different voltages. Mehdi' experiment showed that Lewin's prediction was right. Mehdi admitted that explicitly.

Then Mehdi said that although the experiment matched exactly what Lewin predicted, he, Mehdi, thought that Lewin's was wrong.

This is a case of doublethink, where someone simultaneously accepts two mutually contradictory beliefs as correct, often in contravention to one's own sense of reality.

Quote
One more thing:  bsfeechannel,  I called you out and you didn't deliver. 

Not interested in your piss contest.

Quote
You have no idea of what you are talking about.  Talking to you is exactly like talking to a flat earther or an antivaxer.  So I decided to go thunderf00t on you call out all your BS.

Be my guest.

Hey bsfeechannel, do you have an oscilloscope?   Oh wait, do you even know how to use an oscilloscope?  Do you have any idea of what is the deal with oscilloscope probes?  Did you have an instrumentation course at the university?  I know the answers already: no, no, no, and no.  Because you are a fake!   Your knowledge of these very elemental subjects is minimal at best and shows in your ridiculous comments.

For everyone else reading, who are you going to trust?  Electroboom whom uses multiple oscilloscopes in a regular basis or Lewin that had no idea on what is going on with the oscilloscope and often calls it a voltmeter?



 
Homer: Kids, there's three ways to do things; the right way, the wrong way and the Max Power way!
Bart: Isn't that the wrong way?
Homer: Yeah, but faster!
 

Online bdunham7

  • Super Contributor
  • ***
  • Posts: 7812
  • Country: us
Re: #562 – Electroboom!
« Reply #40 on: November 08, 2021, 03:13:57 am »
For everyone else reading, who are you going to trust?  Electroboom whom uses multiple oscilloscopes in a regular basis or Lewin that had no idea on what is going on with the oscilloscope and often calls it a voltmeter?

Maybe lighten up a bit?  An oscilloscope IS a voltmeter!  And 'trust' isn't the issue, I'm (maybe) interested in what is going on, not comparing the trustworthiness of two rather flamboyant showmen.
A 3.5 digit 4.5 digit 5 digit 5.5 digit 6.5 digit 7.5 digit DMM is good enough for most people.
 
The following users thanked this post: bsfeechannel

Offline jesuscf

  • Frequent Contributor
  • **
  • Posts: 499
  • Country: ca
Re: #562 – Electroboom!
« Reply #41 on: November 08, 2021, 03:38:26 am »
For everyone else reading, who are you going to trust?  Electroboom whom uses multiple oscilloscopes in a regular basis or Lewin that had no idea on what is going on with the oscilloscope and often calls it a voltmeter?

Maybe lighten up a bit?  An oscilloscope IS a voltmeter!  And 'trust' isn't the issue, I'm (maybe) interested in what is going on, not comparing the trustworthiness of two rather flamboyant showmen.

Yes, and oscilloscope measures voltage, but in the profession we don't call it a voltmeter.  If I say: "I used the voltmeter" to measure something, the first thing that comes to your mind is not an oscilloscope for sure!  If you are old enough, not even a multimeter comes to your mind when somebody says 'voltmeter'.  As for lighten up a bit: NO!  Charlatans like bsfeechannel must be confronted.
Homer: Kids, there's three ways to do things; the right way, the wrong way and the Max Power way!
Bart: Isn't that the wrong way?
Homer: Yeah, but faster!
 

Online bdunham7

  • Super Contributor
  • ***
  • Posts: 7812
  • Country: us
Re: #562 – Electroboom!
« Reply #42 on: November 08, 2021, 03:48:22 am »
Very pertinent question. Lewin took that precaution and demonstrated both theoretically and experimentally that the magnetic field outside the solenoid, where its length is much grater than its diameter, is negligible. See his lecture about it [ https://youtu.be/MXuZ1SRjpqk ]. So, from a practical point of view, the field outside the solenoid can be considered zero.

I'll have to look at that. 

Quote
You are assuming your meter doesn't care about path dependence, but numerous experiments show otherwise. In Lewin's experiment, R1 and R2 are subject to electric fields of different intensities (assuming the resistors have the same size). And they can even be calculated. That's why the meters are showing different voltages (assuming there's no varying magnetic field in the loop made by the meter, the probes and the DUT).

If R1 and R2 were connected in parallel and attached to a battery, the voltages would be the same no matter what, because they would be subject to electric fields of the same intensity.

Stop for a moment and think. The voltages between points A and D in Lewin's experiment cannot be the same for the two resistors, otherwise they would have different currents flowing through them, which is impossible.

Without doing a blow by blow response, I'll point out again that there seems to be a lack of clarity about what we agree on and what is in question.

I don't think that there is any dispute that the two resistors will have voltages across them of opposite sign and different magnitude.  Is that point in contention?

It seems fairly clear to me, and it seems to be fairly explicitly admitted in the second video from MIT (perhaps where Lewin got the idea) posted by rfeecs that the 'path dependence' vis a vis the voltmeters is not the path within the loop, but rather the positioning of the test leads going to the voltmeter.  If you want to convince me otherwise, you would have to address my absolute voltmeter thought experiment.  As far as positioning the scope, imagine a two-channel scope positioned well above the apparatus, not to the R1 or R2 side but right between them, far enough away that any electric or magnetic field is truly negligible.  Now everything is identical except the test leads going to the apparatus and their position is the only variable.

As for the rest of the issues and whether the voltages at the resistors must equal the voltage at the test points simply because wires are 'dead shorts' is something that I think involves--as I said previously--another layer of complexity.  'Voltage ambiguity' isn't really a workable concept, thus much of the consternation among the people trying (and mostly failing!) to explain this.  The first video posted by rfeecs seems to bring some concepts in worth looking at, but I can't say I've really considered it enough to comment. 

One thing to consider is your repeated assertion that you can't have a voltage differential in the wires because they have low resistance.  So are there any conditions where that statement is not true?  Are there conditions where a straight piece of wire will have a voltage differential from end-to-end?
A 3.5 digit 4.5 digit 5 digit 5.5 digit 6.5 digit 7.5 digit DMM is good enough for most people.
 

Online bdunham7

  • Super Contributor
  • ***
  • Posts: 7812
  • Country: us
Re: #562 – Electroboom!
« Reply #43 on: November 08, 2021, 03:59:38 am »
NO!  Charlatans like bsfeechannel must be confronted.

Well then help me understand what goes on in the experiment we're debating.

A 3.5 digit 4.5 digit 5 digit 5.5 digit 6.5 digit 7.5 digit DMM is good enough for most people.
 

Offline bsfeechannelTopic starter

  • Super Contributor
  • ***
  • Posts: 1667
  • Country: 00
Re: #562 – Electroboom!
« Reply #44 on: November 08, 2021, 07:21:00 am »
It seems fairly clear to me, and it seems to be fairly explicitly admitted in the second video from MIT (perhaps where Lewin got the idea) posted by rfeecs that the 'path dependence' vis a vis the voltmeters is not the path within the loop, but rather the positioning of the test leads going to the voltmeter.

R1 and R2 establish two different paths. The positioning of the probes is a consequence of not letting the varying magnetic field induce other voltages in the loop that you will have when the probes are attached to the resistors. So there you have your path dependence.

Quote
As far as positioning the scope, imagine a two-channel scope positioned well above the apparatus, not to the R1 or R2 side but right between them, far enough away that any electric or magnetic field is truly negligible.  Now everything is identical except the test leads going to the apparatus and their position is the only variable.

Got it. And that's a very good question. If you position your scope well above the solenoid of instance, with the probes at a right angle with the plane of the circuit, the loop formed by the resistor and the probes will be traversed by the magnetic field. The voltages will be identical. But they will not be the EMF. They'll be of a value between the voltage of both resistors.

Quote
As for the rest of the issues and whether the voltages at the resistors must equal the voltage at the test points simply because wires are 'dead shorts' is something that I think involves--as I said previously--another layer of complexity.

You can get rid of the dead shorts if you attach two resistive wires of different resistances in the same loop configuration. The result will obviously the same.

Quote
'Voltage ambiguity' isn't really a workable concept, thus much of the consternation among the people trying (and mostly failing!) to explain this.

It is not an easy phenomenon to explain because it is unintuitive.

Quote
One thing to consider is your repeated assertion that you can't have a voltage differential in the wires because they have low resistance.  So are there any conditions where that statement is not true? Are there conditions where a straight piece of wire will have a voltage differential from end-to-end?
 
Yes. When the wires are moving perpendicular to the magnetic field.

But in Lewin's circuit the wires are static in relation to the frame of reference. So the voltage across the wires will obey ohms law.

One misconception is that since the secondary of a transformer is just a wire, and I can measure a voltage when I attach a meter to the terminals, it is the wire that is generating the voltage.

What the wire is doing is to set a boundary condition around the loop where the rotational electric field will be forbidden to exist. This will concentrate the field between the terminals. What you are measuring is a voltage produced by an electric field that only exists between the terminals. It doesn't exist in the wire. So it cannot produce a voltage across it.

When you attach a load to the secondary and current flows, then you'll have an electric field inside the wire that will produce a voltage which will be proportional the current times the resistance of the wire. This electric field will have to be discounted from the field at the terminals, because the integral of the field around the loop is proportional to the derivative of the magnetic field ( I.e. the field in the wire plus the field in the load must add up to the induced EMF no matter what).
 

Offline thinkfat

  • Supporter
  • ****
  • Posts: 2150
  • Country: de
  • This is just a hobby I spend too much time on.
    • Matthias' Hackerstübchen
Re: #562 – Electroboom!
« Reply #45 on: November 08, 2021, 08:09:49 am »
I just cannot let this slide:


Quote
One thing to consider is your repeated assertion that you can't have a voltage differential in the wires because they have low resistance.  So are there any conditions where that statement is not true? Are there conditions where a straight piece of wire will have a voltage differential from end-to-end?
 
Yes. When the wires are moving perpendicular to the magnetic field.

But in Lewin's circuit the wires are static in relation to the frame of reference. So the voltage across the wires will obey ohms law.

This is plain nonsense. It doesn't matter if the wires are moving or not. You will have a voltage whenever the magnetic flux changes. There is no term in the Maxwell-Faraday equation that has any component of spatial displacement.

Quote
One misconception is that since the secondary of a transformer is just a wire, and I can measure a voltage when I attach a meter to the terminals, it is the wire that is generating the voltage.

What the wire is doing is to set a boundary condition around the loop where the rotational electric field will be forbidden to exist. This will concentrate the field between the terminals. What you are measuring is a voltage produced by an electric field that only exists between the terminals. It doesn't exist in the wire. So it cannot produce a voltage across it.

But if there is no field "in the wire", how does a Variac work? Or any tapped transformer? Surely the voltage at the tap cannot be due to Ohms law if there's no current flowing? And if you connect the outer terminals of the transformer to a load, does that mean the field inside the wire collapses? Then the voltage at the tap would also collapse. But quite obviously, this isn't the case.

Quote
When you attach a load to the secondary and current flows, then you'll have an electric field inside the wire that will produce a voltage which will be proportional the current times the resistance of the wire. This electric field will have to be discounted from the field at the terminals, because the integral of the field around the loop is proportional to the derivative of the magnetic field ( I.e. the field in the wire plus the field in the load must add up to the induced EMF no matter what).

See above to understand why this cannot be a proper explanation.
Everybody likes gadgets. Until they try to make them.
 

Offline bsfeechannelTopic starter

  • Super Contributor
  • ***
  • Posts: 1667
  • Country: 00
Re: #562 – Electroboom!
« Reply #46 on: November 08, 2021, 09:30:48 am »
This is plain nonsense. It doesn't matter if the wires are moving or not. You will have a voltage whenever the magnetic flux changes. There is no term in the Maxwell-Faraday equation that has any component of spatial displacement.

Nope. You'll not have an electric field inside the wire if it is static with no current flowing through it. If it is moving, the charges in the conductor will experience the Lorentz force and you'll have an electric field inside it.

Quote
But if there is no field "in the wire", how does a Variac work? Or any tapped transformer? Surely the voltage at the tap cannot be due to Ohms law if there's no current flowing? And if you connect the outer terminals of the transformer to a load, does that mean the field inside the wire collapses? Then the voltage at the tap would also collapse. But quite obviously, this isn't the case.

A transformer winding is just a series of loops connected in series. The electric field resides in the space between each turn.

Quote
See above to understand why this cannot be a proper explanation.

See above to understand what is really going on in a transformer.
 

Offline jesuscf

  • Frequent Contributor
  • **
  • Posts: 499
  • Country: ca
Re: #562 – Electroboom!
« Reply #47 on: November 08, 2021, 09:49:55 am »
A transformer winding is just a series of loops connected in series. The electric field resides in the space between each turn.

WHAT!  This quote needs to be preserved for posterity!  :-DD
Homer: Kids, there's three ways to do things; the right way, the wrong way and the Max Power way!
Bart: Isn't that the wrong way?
Homer: Yeah, but faster!
 

Online tszaboo

  • Super Contributor
  • ***
  • Posts: 7359
  • Country: nl
  • Current job: ATEX product design
Re: #562 – Electroboom!
« Reply #48 on: November 08, 2021, 09:54:50 am »
KVL and KCL are special cases of the Maxwell's equations with some assumptions.
Maxwell's equations always hold, if we assume if there is no quantum physics shenanigans.
KVL always hold, if we assume a few things. How to derive it, what to assume? Google it, or open your university books, because in every half decent university they teach this shit, and I am amazed that people still talk about this. I am not surprised if a physics professor in some US university would go ahead and "discover on his own" that KVL suddenly doesn't hold. What I'm amazed that they let this professor teach electronics to students, and had nobody around tell him that "Da! It is in the coursework for the engineers, haven't you read it?"
 

Offline thinkfat

  • Supporter
  • ****
  • Posts: 2150
  • Country: de
  • This is just a hobby I spend too much time on.
    • Matthias' Hackerstübchen
Re: #562 – Electroboom!
« Reply #49 on: November 08, 2021, 11:51:46 am »
A transformer winding is just a series of loops connected in series. The electric field resides in the space between each turn.

WHAT!  This quote needs to be preserved for posterity!  :-DD

Yeah, it's getting more and more interesting with each new post. Now only Lorentz' force creates an electric field but not a changing magnetic flux (despite Maxwell-Faraday telling the opposite), and now you only have an electric field in the space between the transformer windings. I really wonder how antennas ever worked.

Besides, this contradicts his own statement:
Quote
What the wire is doing is to set a boundary condition around the loop where the rotational electric field will be forbidden to exist. This will concentrate the field between the terminals. What you are measuring is a voltage produced by an electric field that only exists between the terminals. It doesn't exist in the wire. So it cannot produce a voltage across it.

If the field "concentrates between the terminals" it should be nowhere else. Not between the windings either, because clearly they are connected to each other and thus there should not be a "boundary condition" where the "rotational electric field" (what?) is "forbidden to exist".

Next argument will be that the field only exists when you connect a meter to measure it, which would violate causality.
Everybody likes gadgets. Until they try to make them.
 


Share me

Digg  Facebook  SlashDot  Delicious  Technorati  Twitter  Google  Yahoo
Smf